4
$\begingroup$

Let $X$ be a hyperelliptic curve and let $i:X\to X$ denote the hyperelliptic involution. Once we fix a point $x_0\in X$ we get the Abel-Jacobi map $AJ:X\to J$ where $J$ denotes the Jacobian variety. Now the Jacobian is also equipped with an involution, namely $x\mapsto x^{-1}$. Is it possible to choose the base point $x_0$ in such a way that the restriction of the involution on the Jacobian is the involution on $X$.

$\endgroup$

2 Answers 2

7
$\begingroup$

Yes, pick $x_0$ to be a Weierstrass point, i.e. a fixed point of the hyperelliptic involution.

Let $\sigma$ denote the hyperelliptic involution on $X$. Under the Abel-Jacobi map we have $x \mapsto [x-x_0]$ and $\sigma(x) \mapsto [\sigma(x) - x_0]$. Now $[x + \sigma(x) - 2x_0]$ is the divisor of a function, since it is the pullback under the hyperelliptic map of a degree zero divisor on $\mathbf P^1$. Hence $AJ(x)$ and $AJ(\sigma(x))$ are inverses.

$\endgroup$
3
$\begingroup$

Another way to think of this is the following (at least over $\mathbb{C}$).

Consider the diagram $$ \begin{array}{ccccc} X & \xrightarrow{AJ \times AJ \circ \sigma} & J \times J & & \\\\ \downarrow & & \downarrow & & \\\\ \mathbb{P}^1 & \to & Sym^2J & \xrightarrow{+} & J \end{array}. $$ Note that the composition along the bottom row must be constant, as there are no non-trivial maps from $\mathbb{P}^1$ to any Abelian variety.

What this tells you is that $f(x) = AJ(x) + AJ\big(\sigma(x)\big)$ is constant. So if you translate it (i.e. choose a different basepoint): $$ \tilde{AJ}(x) = AJ(x) - \frac{f(x)}{2} $$ then you find that the corresponding $\tilde{f}(x) = 0$ for all $x$. That is, the two involutions agree with each other as desired.

$\endgroup$
2
  • $\begingroup$ There seems to be a small argument missing here, since not every translation on $\mathrm{Pic}^0$ comes from a different choice of basepoint. $\endgroup$ May 13, 2011 at 6:55
  • $\begingroup$ If $x_0$ is chosen to be a Weierstrass point then the image of $x_0$ (and hence all of $\mathbb{P}^1$) in $J$ is 0, so there is no need to translate. $\endgroup$
    – Jim Bryan
    May 13, 2011 at 14:40

Your Answer

By clicking “Post Your Answer”, you agree to our terms of service and acknowledge that you have read and understand our privacy policy and code of conduct.

Not the answer you're looking for? Browse other questions tagged or ask your own question.